Martin was selling tickets for a basketball game in a high school. He sold 1,250 tickets and the total amount collected for the game was $2,750. The student tickets cost $2 each, and adult tickets cost $3 each. How many student and adult tickets were sold?

Answers

Answer 1

Answer:

x = number of students tickets = 1,000

y = number of adults tickets = 250

Step-by-step explanation:

Let

x = number of students tickets

y = number of adults tickets

x + y = 1,250 (1)

2x + 3y = 2,750 (2)

Multiply (1) by 2

x + y = 1,250 (1) * 2

2x + 2y = 2,500 (3)

2x + 3y = 2,750 (2)

Subtract (3) from (2) to eliminate x

3y - 2y = 2,750 - 2,500

y = 250

Substitute y = 250 into (1)

x + y = 1,250

x + 250 = 1,250

x = 1,250 - 250

x = 1,000

x = number of students tickets = 1,000

y = number of adults tickets = 250


Related Questions

Hello, Brainly community!

This question is for all of those Calculus people out there.

The volume of a swimming pool is changing with respect to time, such that the volume is given by W(t), where W(t) is measured in cubic centimeters and t is measured in seconds. A tangent line is shown for W(t) at t = 3 seconds. Determine the best estimate for the value of the instantaneous rate of change of W(t) when t = 3.
(I've narrowed down the answer choices to 2, and just really need to find the right way of thinking to find the answer)

(A) W(lim t) as t goes to 3.
(B) [W(3.1) - W(2.9)] / 0.2.

Thank you in advance!

Answers

Answer:

(B)  [tex]\displaystyle \frac{W(3.1) - W(2.9)}{0.2}[/tex]

General Formulas and Concepts:

Calculus

Limits

Derivatives

The definition of a derivative is the slope of the tangent line.

Derivative Notation

Instantaneous Rates

Tangent Line: [tex]\displaystyle f'(x) = \frac{f(b) - f(a)}{b - a}[/tex]

Step-by-step explanation:

Since we are trying to find a rate at which W(t) changes, we must find the derivative at t = 3.

We are given 2 close answer choices that would have the same numerical answer but different meanings:

(A)  [tex]\displaystyle \lim_{t \to 3} W(t)[/tex](B)  [tex]\displaystyle \frac{W(3.1) - W(2.9)}{0.2}[/tex]

If we look at answer choice (A), we see that our units would simply just be volume. It would not have the units of a rate of change. Yes, it may be the closest numerically correct answer, but it does not tell us the rate at which the volume would be changing and it is not a derivative.

If we look at answer choice (B), we see that our units would be cm³/s, and that is most certainly a rate of change. Answer choice (B) is also a derivative at t = 3, and a derivative tells us what rate something is changing.

∴ Answer choice (B) will give us the best estimate for the value of the instantaneous rate of change of W(t) when t = 3.

Topic: AP Calculus AB/BC (Calculus I/I + II)

Unit: Differentiation

Book: College Calculus 10e

How many black berry trees were sampled to create this graph

Answers

Answer:

C. 31 trees

Step-by-step explanation:

Add up all the frequencies: 3+3+8+10+5+2=31

Hope this helps!!!

Answer:

It's C. 31 trees

Step-by-step explanation:

That's about all I can say

Can someone help with problems fivethroughseven

Answers

Answer:

5) 15120

6) 11880

7) 336

Step-by-step explanation:

The formula for permutation where mPn is m!/(m-n)!

Applying this to question 5, we get 9!/4!, which is 15120.

For question 6, we get 12!/8!, which is 11880.

For question 7, we get 8!/5!, which is 336.

Quadrilateral A B C D is shown. The uppercase right angle, angle A, is 79 degrees.
What are the remaining angle measures if the figure is to be a parallelogram?

m∠B =
°

m∠C =
°

m∠D =
°

Answers

Answer:

m∠B =  

✔ 101

°

m∠C =  

✔ 79

°

m∠D =  

✔ 101

°

Step-by-step explanation:

Answer:

The answer above is right!

The correct answers are:

First box: option C. 101

Second box: option B. 79

Third box: option C. 101

Step-by-step explanation:

Just got it right on edge - Hope it helps :)

Brainliest would be greatly appreciated :D

These points are linear.
Find the slope.
x-3 -2 -1 0 1/2
y-3 -2 -1 0 1 2
slope = [?]

Answers

Answer:

Slope = 4

Step-by-step explanation:

The slope of a linear equation can be calculated as;

m = y2-y1/x2-x1

What we have to do here is to select any two points ;

(x1,y1) = (3,8)

(x2,y2) = (6,20)

m = (20-8)/(6-3) = 12/3 = 4

what do you understand by socialization​

Answers

Socialization is the lifelong process through which people learn the values and norms of a given society.

URGENT PLS ANSWER QUICKLY​

Answers

Answer:

9th is 44 mark me as brainlist

Answer:

Question 9 is perimeter

Solve for the questions (both of them) and label you answers for which question

Answers

Jjbbh I don’t under stand

PLZ HELP its due soon!!!

Answers

Answer:

E)

Step-by-step explanation:

1) 6.50 (starting with his current wage)

2) 6.50 + 0.25 = 6.75 $

3) 6.75 + 0.25 = 7.00 $

4) 7.00 + 0.25 = 7.25 $

5) 7.25 + 0.25 = 7.50 $

6) 7.50 + 0.25 = 7.75 $

the red line below is perpendicular to which of the following

Answers

The red line is perpendicular to the y-axis.

Shirley buys fiction books for $20 each, and then marks up by 25% to
resell. What is the markup in dollars?

Answers

Answer:

$5

Step-by-step explanation:

Find the markup by finding 25% of 20:

20(0.25)

= 5

So, the markup is $5

Which of the following is most likely the next step in the series?
А.
B.
C.
D.

Answers

Answer:

D

Step-by-step explanation:

The answer is D because the pattern adds one angle every one shape. Ex. Triangle, Rectangle, Pentagon, Hexagon

Solve for x and y:
x – 3y = -8
3x + 2y = 31
Select one:
O a. (-11,-1)
O b. (11,1)
o c. (5,8)
O d. (7,5)

Answers

Answer:

O d. (7,5)

Step-by-step explanation:

x – 3y = -8 (1)

3x + 2y = 31 (2)

From (1)

x = -8 + 3y

Substitute x = -8 + 3y into (2)

3x + 2y = 31 (2)

3(-8 + 3y) + 2y = 31

-24 + 9y + 2y = 31

-24 + 11y = 31

11y = 31 + 24

11y = 55

y = 55/11

y = 5

Substitute y = 5 into (1)

x – 3y = -8 (1)

x - 3(5) = -8

x - 15 = -8

x = -8 + 15

x = 7

(x, y) = (7, 5)

Find the value of x from the following given figures.​

Answers

solution :-

here,

We know that interior opposite angles are equal.

So,

110° = 50° + x (being interior opposite angles)

110° - 50° = x

60° = x

the value of x =60°

hope it is helpful to you ☺️

PLS HELP ASAP, I need it in 10 mins. I GIVE 15 PTS !!!! if v1 = (3,-4) and v2 = (2,6) then v1*v2 is equal to which of the following?
A. 30
B. (-12, -24)
C. (6,-24)
D. -18

Answers

It is C. Good luck on ur test!!!!

v₁·v₂ is -18 which is correct option(D)

What are Arithmetic operations?

Arithmetic operations can also be specified by the subtract, divide, and multiply built-in functions.

The operator that perform arithmetic operation are called arithmetic operators .

Operators which let do basic mathematical calculations

+ Addition operation : Adds values on either side of the operator.

For example 4 + 2 = 6

- Subtraction operation : Subtracts right hand operand from left hand operand.

for example 4 -2 = 2

* Multiplication operation : Multiplies values on either side of the operator

For example 4*2 = 8

/ Division operation : Divides left hand operand by right hand operand

For example 4/2 = 2

Given that,

v1 = (3,-4) and v2 = (2,6)

To determine v₁·v₂

v₁·v₂ = 3·(2) - 4·(6)

v₁·v₂ = 6 - 24

v₁·v₂ = -18

Hence, the v₁·v₂ is -18.

Learn more about Arithmetic operations here:

brainly.com/question/25834626

#SPJ2

Which number completes the system of linear
inequalities represented by the graph?
y> 2x – 2 and x + 4y > _____

Answers

The system of linear inequalties that is represented by the graph is formed by the expressions y > 2 · x - 2 and x + 4 · y > - 12.

What system of inequalities generate the graph seen in the image?

Herein we have two inequalities generated by two linear functions whose form is described below:

f(x, y) > a (blue region)      (1)

g(x, y) > b (red region)      (2)

By direct comparison, we find that the limit of the red region is represented by the inequality y > 2 · x - 2. From the equation of the line we can derive an expression for the limit of the blue region:

Slope

m = [- 4 - (- 3)]/(4 - 0)

m = - 1/4

Intercept

b = y - m · x

b = - 3 - (- 1/4) · 0

b = - 3

Then, the inequation is represented by the linear equation:

y > - (1/4) · x - 3

4 · y > - x - 12

x + 4 · y > - 12

The system of linear inequalties that is represented by the graph is formed by the expressions y > 2 · x - 2 and x + 4 · y > - 12.

To learn more on inequalities: https://brainly.com/question/20383699

#SPJ1

If anyone can help with this ill mark Brainly

Answers

Answer:

In this case, we can do substitution.

Step-by-step explanation:

For the first one, (s - t)(x) = ((x - 5) - 4x^2)(x) = x^2 - 21x

For the second one, (s*t)(x) = ((x - 5) *4x^2)(x) = 4x^4 - 20x^3

And for the last one,  (s+t)(-2) = ((x - 5) + 4x^2)(-2) = -8x^2 - 2x + 10

Hope your happy with the answer :)

Find the equation of a line that is perpendicular to y = -3x – 1 and passes through the point
(3,2).
Give your answer in the form y = mx + b.

Answers

the answer is:
y = 1/3x + 1
I think the answer is y= ⅓ + 1

A line with a slope of 3 passes through the point (-1, 2).

Write an equation for this line in point-slope form.

Answers

Answer:

Step-by-step explanation:

Slope = m = 3

(x₁ , y₁) = (-1 , 2)

Point slope form: y - y₁ = m(x - x₁)

y - 2 = 3(x - [-1] )

y - 2 = 3(x + 1)

y - 2 = 3*x + 3*1

y - 2 = 3x + 3

    y = 3x + 3 + 2

    y = 3x + 5

Help!!!
Find the domain of the function. Write the answer in interval notation.

Answers

Answer:

A

Step-by-step explanation:

Given

f(x) = [tex]\frac{2}{3x+2}[/tex]

The denominator cannot be zero as this would make f(x) undefined.

Equating the denominator to zero and solving gives the value that x cannot be.

3x + 2 = 0 ⇒ 3x = - 2 ⇒ x = - [tex]\frac{2}{3}[/tex] ← excluded value

Then

domain is ( - ∞ , - [tex]\frac{2}{3}[/tex] ) U ( - [tex]\frac{2}{3}[/tex], ∞ ) → A

Graph the solution of the inequality 3/7(35x-14)<_ 21x/2+3

Answers

Answer:

You'll have a closed circle at x = 2, and shading to the left

See the diagram below

=========================================================

Explanation:

The fractions here are 3/7 and 21/2. The denominators of which are 7 and 2 respectively. The LCD is 7*2 = 14.

If we multiply both sides by 14, then this will clear out the denominators and make the fractions go away.

14*(3/7) = (14*3)/7 = 42/7 = 814*(21/2) = (14*21)/2 = 294/2 = 147

So if we multiplied both sides by 14, then we have these steps

[tex]\frac{3}{7}(35x-14) \le \frac{21x}{2}+3\\\\14*\frac{3}{7}(35x-14) \le 14*\left(\frac{21x}{2}+3\right)\\\\14*\frac{3}{7}(35x-14) \le 14*\left(\frac{21x}{2}\right)+14*\left(3\right)\\\\6(35x-14) \le 147x+42\\\\[/tex]

--------------------------

Let's isolate x

[tex]6(35x-14) \le 147x+42\\\\6(35x)+6(-14) \le 147x+42\\\\210x-84 \le 147x+42\\\\210x-147x \le 42+84\\\\63x \le 126\\\\x \le 126/63\\\\x \le 2\\\\[/tex]

The graph of this will consist of a closed or filled in circle at x = 2. We shade to the left to represent numbers smaller than 2.

So either x = 2 or x < 2.

If we used an open hole at 2, then we wouldn't be including 2 (but we want to include this endpoint).

See the diagram below.

Y=2/7 -7 y=-x+2 What is the solution for this system of equations?​

Answers

Answer:

(61/7, -47/7)

Step-by-step explanation:

Given: y=2/7 -7

           y=-x+2

Rewrite the top equation ( y=2/7 -7

y = -47/7

y = -x + 2

Because both equations are equal to y, we can rewrite it again.

-47/7 = -x + 2

Add x to both sides

x + (-47/7) = 2

Add 47/7 to both sides.

x = 47/7 + 2/1

x = 61/7

As stated in the beginning, y is equal to -47/7

Hope you understand!

Drag


the yellow point until an accurate "height" of the triangle is drawn. Afterwards, fill out the empty boxes below to determine the area of the triangle.

Answers

Answer:

I don't see a yellow point

NEED HELPPPP RNNNNNNNN​

Answers

put it together first so
x+12-5x=3 solve from there it easy

Which is the area between the x-axis and y=x from x=1 to x=5

Answers

Answer:

[tex]\displaystyle A = 12[/tex]

General Formulas and Concepts:

Pre-Algebra

Order of Operations: BPEMDAS

BracketsParenthesisExponentsMultiplicationDivisionAdditionSubtractionLeft to Right

Algebra I

FunctionsFunction NotationGraphing

Calculus

Integrals

Definite IntegralsArea under the curve

Integration Rule [Reverse Power Rule]:                                                               [tex]\displaystyle \int {x^n} \, dx = \frac{x^{n + 1}}{n + 1} + C[/tex]

Integration Rule [Fundamental Theorem of Calculus 1]:                                     [tex]\displaystyle \int\limits^b_a {f(x)} \, dx = F(b) - F(a)[/tex]

Area of a Region Formula:                                                                                     [tex]\displaystyle A = \int\limits^b_a {[f(x) - g(x)]} \, dx[/tex]

Step-by-step explanation:

Step 1: Define

Identify

y = x

Interval: x = 1 to x = 5

Step 2: Sort

Graph the function. See Attachment.

Bounds of Integration: [1, 5]

Step 3: Find Area

Substitute in variables [Area of a Region Formula]:                                   [tex]\displaystyle A = \int\limits^5_1 {x} \, dx[/tex][Integral] Integrate [Integration Rule - Reverse Power Rule]:                     [tex]\displaystyle A = \frac{x^2}{2} \bigg| \limits^5_1[/tex]Evaluate [Integration Rule - Fundamental Theorem of Calculus 1]:           [tex]\displaystyle A = 12[/tex]

Topic: AP Calculus AB/BC (Calculus I/I + II)

Unit: Integration

Book: College Calculus 10e

Ava’s gross pay is $2,500 per month. Her deductions include the following:

Federal income tax $290

State income tax $85

Social Security $112

Medicare $35

What is Ava’s net pay each month?

Answers

$1,978

Explanation:

Gross pay is what’s made before taxes and necessary deductions, net pay is after taxes and necessary deductions. So we minus the deductions to find the net pay. The equation is 2,500 - 290 - 85 - 112 - 35 = ? . The answer is $1,978.

I hope this helps. Please mark me the Brainliest, it’s not necessary but I put time and effort into every answer and I would appreciate it greatly. Have a great day, stay safe and stay healthy ! :)

CAN SOMEBODY PLEASE HELP MEEEE

Answers

Answer:

105.

Step-by-step explanation:

.

PLEASE HELP ASAP WHAT IS THE ANSWER TO THIS QUADRATIC EQUATIONNNN

Answers

Answer:

pls i don't know

The answer is A. 2

This is because the stretch of the parabola is elongated by 2!

Which of the following is not a way to represent the solution of the inequality 2(x − 1) greater than or equal to −12? (1 point)

x greater than or equal to −5
x less than or equal to −5
−5 less than or equal to x
A number line with a closed circle on negative 5 and shading to the right

Answers

Answer:

x less than or equal to -5

Step-by-step explanation:

2(x-1) ≥ -12

2x - 2 ≥ -12

2x ≥ -10

x ≥ -5

Find mCFB

Help me please

Answers

Answer:

A

Step-by-step explanation:

<CPA = 50o                     Given

<FPB = 50o                      Vertically opposite a 50 degree angle

<CPF = 90o                      Given

<CPB = <FPB + <CPF       Given or found

<CPB = 50 + 90                Substitute

<CPB = 140o

Other Questions
what caused the trend tords city life in the 1800's how does high death rate affect population and development in a country Match the reaction to the type that best describes it.2HCl H2 + Cl2?CombinationNaF + KBr NaBr + KE?DecompositionMg + MnCl2 -Mn + MgCl2?Single replacement2Na + 02 Na2O2?Double replacement Which operation will solve the following word problem? Jeff earns $14.00 per hour, Tom earns half as much as Jeff. How much does Tom earn per hour?MultiplicationSubtractionAdditionDivision Describe about comparative cost and absolute advantages of international trade Revenue is recorded when services have been performed or products have been delivered to customers. The accounting principle supporting this reporting is Jenna has just finished gathering information to write a paper for her English composition class. She begins writing using information from books and articles on her topic. What strategy could Jenna use that would help her avoid plagiarism Answer with explaination every steps The number of periods/series in Mendeleev's Periodic table isA. 10B. 13C. 12D. 14 Mary is 3 years older than Sarah. Winifred is twice as old as Mary. Altogether their ages total 89. How old is Sarah? 24 years old 22 years old 18 years old None of these choices are correct. A 6.0-cm-diameter horizontal pipe gradually narrows to 4.0 cm. When water flows through this pipe at a certain rate, the gauge pressure in these two sections is 32.0 kPa and 24.0 kPa, respectively. What is the volume rate of flow? What is the value of x in the equation 0.7x 1.4 = 3.5?1. -72. -33. 74. 3 Use Cramer's Rule to solve (if possible) the system of linear equations. x1 + 2x2 =8- x1 + x2 = 1Required:Find the coefficient matrix. Find the product. If the result is negative, enter "-". If the result is positive, enter "+".-7(- a2 ) 2 ( -b3 ) What is the net ionic equation for the following? :)1. Mg (s) + Na2CrO4 (aq) -->2. K (s) + Cd(NO3)2 (aq) --> Help ASAP Iill mark you as brainlister Equation for the length of a rectangle is two more than three times the width area is 161 The phase of the business cycle with the highest level of GDP and the lowest unemployment rate is called an? Helpp m and explain ,I will mark brainlest:) please tell me the answer to this, thanks